Stay ahead of learning milestones! Enroll in a class over the summer!

G
Topic
First Poster
Last Poster
Feet of perpendiculars to diagonal in cyclic quadrilateral
jl_   1
N an hour ago by navier3072
Source: Malaysia IMONST 2 2023 (Primary) P6
Suppose $ABCD$ is a cyclic quadrilateral with $\angle ABC = \angle ADC = 90^{\circ}$. Let $E$ and $F$ be the feet of perpendiculars from $A$ and $C$ to $BD$ respectively. Prove that $BE = DF$.
1 reply
jl_
2 hours ago
navier3072
an hour ago
IMO Shortlist 2014 N5
hajimbrak   58
N an hour ago by Jupiterballs
Find all triples $(p, x, y)$ consisting of a prime number $p$ and two positive integers $x$ and $y$ such that $x^{p -1} + y$ and $x + y^ {p -1}$ are both powers of $p$.

Proposed by Belgium
58 replies
+1 w
hajimbrak
Jul 11, 2015
Jupiterballs
an hour ago
Integer a_k such that b - a^n_k is divisible by k
orl   69
N an hour ago by ZZzzyy
Source: IMO Shortlist 2007, N2, Ukrainian TST 2008 Problem 10
Let $b,n > 1$ be integers. Suppose that for each $k > 1$ there exists an integer $a_k$ such that $b - a^n_k$ is divisible by $k$. Prove that $b = A^n$ for some integer $A$.

Author: Dan Brown, Canada
69 replies
orl
Jul 13, 2008
ZZzzyy
an hour ago
interesting function equation (fe) in IR
skellyrah   1
N an hour ago by CrazyInMath
Source: mine
find all function F: IR->IR such that $$ xf(f(y)) + yf(f(x)) = f(xf(y)) + f(xy) $$
1 reply
skellyrah
3 hours ago
CrazyInMath
an hour ago
Find maximum area of right triangle
jl_   1
N an hour ago by navier3072
Source: Malaysia IMONST 2 2023 (Primary) P4
Given a right-angled triangle with hypothenuse $2024$, find the maximal area of the triangle.
1 reply
jl_
2 hours ago
navier3072
an hour ago
Erasing a and b and replacing them with a - b + 1
jl_   1
N 2 hours ago by maromex
Source: Malaysia IMONST 2 2023 (Primary) P5
Ruby writes the numbers $1, 2, 3, . . . , 10$ on the whiteboard. In each move, she selects two distinct numbers, $a$ and $b$, erases them, and replaces them with $a+b-1$. She repeats this process until only one number, $x$, remains. What are all the possible values of $x$?
1 reply
jl_
2 hours ago
maromex
2 hours ago
Prove that sum of 1^3+...+n^3 is a square
jl_   2
N 2 hours ago by NicoN9
Source: Malaysia IMONST 2 2023 (Primary) P1
Prove that for all positive integers $n$, $1^3 + 2^3 + 3^3 +\dots+n^3$ is a perfect square.
2 replies
jl_
2 hours ago
NicoN9
2 hours ago
x^3+y^3 is prime
jl_   2
N 2 hours ago by Jackson0423
Source: Malaysia IMONST 2 2023 (Primary) P3
Find all pairs of positive integers $(x,y)$, so that the number $x^3+y^3$ is a prime.
2 replies
jl_
2 hours ago
Jackson0423
2 hours ago
EGMO magic square
Lukaluce   16
N 2 hours ago by zRevenant
Source: EGMO 2025 P6
In each cell of a $2025 \times 2025$ board, a nonnegative real number is written in such a way that the sum of the numbers in each row is equal to $1$, and the sum of the numbers in each column is equal to $1$. Define $r_i$ to be the largest value in row $i$, and let $R = r_1 + r_2 + ... + r_{2025}$. Similarly, define $c_i$ to be the largest value in column $i$, and let $C = c_1 + c_2 + ... + c_{2025}$.
What is the largest possible value of $\frac{R}{C}$?

Proposed by Paulius Aleknavičius, Lithuania
16 replies
Lukaluce
Apr 14, 2025
zRevenant
2 hours ago
A colouring game on a rectangular frame
Tintarn   1
N 2 hours ago by pi_quadrat_sechstel
Source: Bundeswettbewerb Mathematik 2025, Round 1 - Problem 4
For integers $m,n \ge 3$ we consider a $m \times n$ rectangular frame, consisting of the $2m+2n-4$ boundary squares of a $m \times n$ rectangle.

Renate and Erhard play the following game on this frame, with Renate to start the game. In a move, a player colours a rectangular area consisting of a single or several white squares. If there are any more white squares, they have to form a connected region. The player who moves last wins the game.

Determine all pairs $(m,n)$ for which Renate has a winning strategy.
1 reply
Tintarn
Mar 17, 2025
pi_quadrat_sechstel
2 hours ago
Sum and product of 5 numbers
jl_   0
2 hours ago
Source: Malaysia IMONST 2 2023 (Primary) P2
Ivan bought $50$ cats consisting of five different breeds. He records the number of cats of each breed and after multiplying these five numbers he obtains the number $100000$. How many cats of each breed does he have?
0 replies
jl_
2 hours ago
0 replies
Geometry Finale: Incircles and concurrency
lminsl   173
N Apr 4, 2025 by Parsia--
Source: IMO 2019 Problem 6
Let $I$ be the incentre of acute triangle $ABC$ with $AB\neq AC$. The incircle $\omega$ of $ABC$ is tangent to sides $BC, CA$, and $AB$ at $D, E,$ and $F$, respectively. The line through $D$ perpendicular to $EF$ meets $\omega$ at $R$. Line $AR$ meets $\omega$ again at $P$. The circumcircles of triangle $PCE$ and $PBF$ meet again at $Q$.

Prove that lines $DI$ and $PQ$ meet on the line through $A$ perpendicular to $AI$.

Proposed by Anant Mudgal, India
173 replies
lminsl
Jul 17, 2019
Parsia--
Apr 4, 2025
Geometry Finale: Incircles and concurrency
G H J
Source: IMO 2019 Problem 6
The post below has been deleted. Click to close.
This post has been deleted. Click here to see post.
lminsl
544 posts
#1 • 79 Y
Y by babu2001, JustKeepRunning, Illuzion, Wizard_32, TheMathLife, anantmudgal09, Mr.Chagol, e_plus_pi, char2539, TheDarkPrince, MathInfinite, PCM_noob, FadingMoonlight, MathPassionForever, Euler1728, rmtf1111, Davrbek, Generic_Username, Fermat_Theorem, samuel, Carpemath, Mathman12334, wx_family2, Kayak, fjm30, buratinogigle, v_Enhance, bel.jad5, rkm0959, Satyanweshi_math1729, HolyMath, GeoMetrix, Drunken_Master, SHREYAS333, mathleticguyyy, Arhaan, amar_04, Supercali, EulersTurban, lilavati_2005, Polynom_Efendi, Aryan-23, Feridimo, NJOY, Pi-is-3, AmirKhusrau, kc5170, Purple_Planet, donfluffles1412, Atpar, Inconsistent, IAmTheHazard, psi241, srijonrick, Bassiskicking, OlympusHero, Kanep, TheMathCruncher_007, sotpidot, megarnie, centslordm, tiendung2006, rayfish, magicarrow, crazyeyemoody907, GeoKing, Jalil_Huseynov, mathmax12, EpicBird08, Lamboreghini, Adventure10, NO_SQUARES, CrimsonBlade273, Rounak_iitr, Eka01, Sedro, Funcshun840, iamnotgentle, wizixez
Let $I$ be the incentre of acute triangle $ABC$ with $AB\neq AC$. The incircle $\omega$ of $ABC$ is tangent to sides $BC, CA$, and $AB$ at $D, E,$ and $F$, respectively. The line through $D$ perpendicular to $EF$ meets $\omega$ at $R$. Line $AR$ meets $\omega$ again at $P$. The circumcircles of triangle $PCE$ and $PBF$ meet again at $Q$.

Prove that lines $DI$ and $PQ$ meet on the line through $A$ perpendicular to $AI$.

Proposed by Anant Mudgal, India
This post has been edited 2 times. Last edited by djmathman, Jul 17, 2019, 12:31 PM
Z K Y
The post below has been deleted. Click to close.
This post has been deleted. Click here to see post.
babu2001
402 posts
#2 • 86 Y
Y by Ankoganit, djmathman, Kayak, RAMUGAUSS, math_pi_rate, Illuzion, Wizard_32, AngleChasingXD, div5252, MathPassionForever, Pluto1708, e_plus_pi, Ygg, PhysicsMonster_01, ELMOliveslong, Delta0001, TheDarkPrince, rkm0959, ssk9208, wu2481632, Vlados021, AlastorMoody, MathInfinite, The_Maitreyo1, FadingMoonlight, EulerMacaroni, Euler1728, RudraRockstar, pad, anantmudgal09, Kagebaka, Vietjung, Mathman12334, Carpemath, fjm30, JustKeepRunning, buratinogigle, Ultroid999OCPN, dbs27, SHREYAS333, mathleticguyyy, Arhaan, amar_04, Supercali, myh2910, Aryan-23, Pi-is-3, AmirKhusrau, Understandingmathematics, GeoMetrix, Bumblebee60, Williamgolly, Toinfinity, StagerNo4, Purple_Planet, donfluffles1412, mijail, Inconsistent, Dr_Vex, Hamroldt, Bassiskicking, OlympusHero, TheMathCruncher_007, Rg230403, CaptainLevi16, Pratik12, centslordm, megarnie, CyclicISLscelesTrapezoid, rayfish, Jasurbek, PRMOisTheHardestExam, magicarrow, crazyeyemoody907, michaelwenquan, mathmax12, EpicBird08, Lamboreghini, Adventure10, Mango247, Sedro, ehuseyinyigit, Funcshun840, iamnotgentle, MS_asdfgzxcvb, farhad.fritl
This problem was proposed by Anant Mudgal, India
Z K Y
The post below has been deleted. Click to close.
This post has been deleted. Click here to see post.
tastymath75025
3223 posts
#3 • 37 Y
Y by nguyendangkhoa17112003, Generic_Username, chih, karitoshi, FadingMoonlight, Pathological, Supercali, anantmudgal09, Kassuno, StoryScene, Vietjung, tapir1729, cowcow, Carpemath, pinetree1, GeoMetrix, BobaFett101, myh2910, Aryan-23, Polynom_Efendi, 507219, AmirKhusrau, Understandingmathematics, Purple_Planet, donfluffles1412, Bassiskicking, OlympusHero, TheMathCruncher_007, math31415926535, centslordm, rayfish, ike.chen, EpicBird08, mathmax12, Adventure10, Sedro, Rounak_iitr
Nice problem :)

First note $\angle BQC=\angle BQP+\angle CQP = \angle BFP+\angle CEP = \frac{1}{2}\overarc{EDF} = \angle BIC$ so $BQIC$ is cyclic. Now let $PQ\cap (BIC) = Y$. The main claim is that $FREP, BICY$ are similar. Both quadrilaterals are cyclic, so it suffices to show the measures of the corresponding arcs on the two circles are equal. Clearly $\overarc{FR}, \overarc{BI}$ are both equal to $\angle C$, and similarly $\overarc{RE},\overarc{CI}$ are also equal. Also, $\overarc{FP} = 2\angle BFP=2\angle BQP = \overarc{BY}$ and similarly $\overarc{EP}=\overarc{CY}$, so $FREP, BICY$ are similar.

Now we ignore $Q$ and let $X\in DI$ with $AX\perp AI$; it suffices to show $P,X,Y$ are collinear. Let $D'$ be the antipode to $D$ on $\omega$ and $A_1,B_1,C_1$ be the midpoints of $EF,DF,DE$. Note that $(R,P;E,F)$ is obviously harmonic, so similarity implies $(B,C;I,Y)$ is too. It's well-known that $D',M,P$ are collinear (say, by projecting harmonic bundle $(R,P;E,F)$ through $D'$ onto $EF$). Meanwhile, inversion with respect to $\omega$ sends $B,C$ to $B_1,C_1$ and sends $I$ to infinity, so to preserve the harmonic bundle $(B,C;I,Y)$ the inversion must send $Y$ to the midpoint $Y'$ of $B_1C_1$. Finally, let $X'$ be the foot of the projection from $A_1$ to $DI$, so clearly $X',X$ correspond in the inversion.

Now after inversion we wish to show $P,Y',X',I$ are concyclic. Let $D_1 = DA_1\cap \omega$; clearly $D_1D'X'A_1$ is cyclic with diameter $A_1D'$, so $DA_1\cdot DD_1 = DX'\cdot DD'$. Since $I,Y'$ are midpoints of $DD', DA_1$ this implies $DI\cdot DX' = DY'\cdot DD_1$ so $D_1\in (IX'Y')$. Now note that $IY'||D'P$ so $\angle PIY' = \angle IPD' = \angle ID'P = \angle DD'P =\angle DD_1P=\angle Y'D_1P$, so $P\in (D_1IX'Y')$ and we're done.
This post has been edited 3 times. Last edited by tastymath75025, Jul 17, 2019, 12:54 PM
Z K Y
The post below has been deleted. Click to close.
This post has been deleted. Click here to see post.
Mr.Chagol
249 posts
#4 • 14 Y
Y by MywifeLQH, Mathman12334, Hexagrammum16, Aryan-23, RudraRockstar, AmirKhusrau, Purple_Planet, TheMathCruncher_007, megarnie, Lamboreghini, Adventure10, Mango247, Funcshun840, anantmudgal09
So congrats Anant for this great prob
Z K Y
The post below has been deleted. Click to close.
This post has been deleted. Click here to see post.
v_Enhance
6874 posts
#7 • 54 Y
Y by Tuleuchina, CeuAzul, nguyendangkhoa17112003, chih, The_Maitreyo1, FadingMoonlight, ETS1331, turko.arias, mykwok, mathisawesome2169, anantmudgal09, Lol_man000, Carpemath, Vietjung, cowcow, Mathman12334, AMN300, Flash_Sloth, Pluto1708, yugrey, GammaBetaAlpha, Batred625, rkm0959, Ultroid999OCPN, GeoMetrix, myh2910, Loppukilpailija, Aryan-23, neel02, Cindy.tw, Pi-is-3, 507219, AmirKhusrau, Purple_Planet, Atpar, Gaussian_cyber, v4913, TheMathCruncher_007, math31415926535, centslordm, HamstPan38825, rayfish, crazyeyemoody907, Quidditch, lahmacun, mathmax12, EpicBird08, Zfn.nom-_nom, Lamboreghini, Adventure10, sabkx, Sedro, Rounak_iitr, wizixez
We use complex numbers with $D=x$, $E=y$, $F=z$. Then $A = \frac{2yz}{y+z}$, $R = \frac{-yz}{x}$ and so \[ P = \frac{A-R}{1-R\overline{A}} 	= \frac{\frac{2yz}{y+z} + \frac{yz}{x}} 	{1 + \frac{yz}{x} \cdot \frac{2}{y+z}} 	= \frac{yz(2x+y+z)}{2yz+x(y+z)}. \]We now compute \begin{align*} 	O_B &= \det \begin{bmatrix} 		P & P \overline P & 1 \\ 		F & F \overline F & 1 \\ 		B & B \overline B & 1 	\end{bmatrix} 	\div \det \begin{bmatrix} 		P & \overline P & 1 \\ 		F & \overline F & 1 \\ 		B & \overline B & 1 	\end{bmatrix} 	= \det \begin{bmatrix} 		P & 1 & 1 \\ 		z & 1 & 1 \\ 		\frac{2xz}{x+z} & \frac{4xz}{(x+z)^2} & 1 	\end{bmatrix} 	\div \det \begin{bmatrix} 		P & 1/P & 1 \\ 		z & 1/z & 1 \\ 		\frac{2xz}{x+z} & \frac{2}{x+z} & 1 	\end{bmatrix} \\ 	&= \frac{1}{x+z} \det \begin{bmatrix} 		P & 0 & 1 \\ 		z & 0 & 1 \\ 		2xz(x+z) & -(x-z)^2 & (x+z)^2 	\end{bmatrix} 	\div \det \begin{bmatrix} 		P & 1/P & 1 \\ 		z & 1/z & 1 \\ 		2xz & 2 & x+z 	\end{bmatrix} \\ 	&= \frac{(x-z)^2}{x+z} \cdot \frac{P-z}{(x+z)(P/z-z/P)+2z-2x + \frac{2xz}{P}-2P} \\ 	&= \frac{(x-z)^2}{x+z} \cdot \frac{P-z}{ 		(\frac xz-1) P - 2(x-z) + (xz-z^2) \frac 1P  } \\ 	&= \frac{x-z}{x+z} \cdot \frac{P-z}{P/z + z/P - 2} 	= \frac{x-z}{x+z} \cdot \frac{P-z}{\frac{(P-z)^2}{Pz}} 	= \frac{x-z}{x+z} \cdot \frac{1}{\frac 1z - \frac 1P} \\ 	&= \frac{x-z}{x+z} \cdot \frac{y(2x+y+z)}{y(2x+y+z) - (2yz+xy+xz)} 	= \frac{x-z}{x+z} \cdot \frac{yz(2x+y+z)}{xy+y^2-yz-xz} \\ 	&= \frac{x-z}{x+z} \cdot \frac{yz(2x+y+z)}{(y-z)(x+y)}. \end{align*}Similarly \[ O_C = \frac{x-y}{x+y} \cdot \frac{yz(2x+y+z)}{(z-y)(x+z)}. \]Therefore, subtraction gives \[ O_B-O_C 	= 	\frac{yz(2x+y+z)}{(x+y)(x+z)(y-z)} 	\left[ (x-z)+(x-y) \right] 	= \frac{yz(2x+y+z)(2x-y-z)}{(x+y)(x+z)(z-y)}. \]It remains to compute $T$. Since $T \in \overline{ID}$ we have $t/x \in {\mathbb R}$ so $\overline t = t/x^2$. Also, \begin{align*} 	\frac{t - \frac{2yz}{y+z}}{y+z} \in i {\mathbb R} 	\implies 0 &= \frac{t-\frac{2yz}{y+z}}{y+z} 	+ \frac{\frac{t}{x^2}-\frac{2}{y+z}}{\frac1y+\frac1z} \\ 	&= \frac{1+\frac{yz}{x^2}}{y+z} t - \frac{2yz}{(y+z)^2} - \frac{2yz}{(y+z)^2} \\ 	\implies t &= \frac{x^2}{x^2+yz} \cdot \frac{4yz}{y+z} \end{align*}Thus \begin{align*} 	P-T &= \frac{yz(2x+y+z)}{2yz+x(y+z)} - \frac{4x^2yz}{(x^2+yz)(y+z)} \\ 	&= yz \cdot \frac{(2x+y+z)(x^2+yz)(y+z) - 4x^2(2yz+xy+xz)} 		{(y+z)(x^2+yz)(2yz+xy+xz)} \\ 	&= -yz \cdot \frac{(2x-y-z)(x^2y+x^2z+4xyz+y^2z+yz^2)} 		{(y+z)(x^2+yz)(2yz+xy+xz)}. \end{align*}This gives $\overline{PT} \perp \overline{O_B O_C}$ as needed.
This post has been edited 3 times. Last edited by v_Enhance, Jul 20, 2019, 8:48 PM
Z K Y
The post below has been deleted. Click to close.
This post has been deleted. Click here to see post.
Systematicworker
83 posts
#8 • 2 Y
Y by Adventure10, ehuseyinyigit
Is there any solution using famous lemmas related to incircle and circumcircle?
Z K Y
The post below has been deleted. Click to close.
This post has been deleted. Click here to see post.
JND
658 posts
#12 • 1 Y
Y by Adventure10
Last year, there was also a Geometry problem on P6 of the day 2. Similarly, This year, there is also another geometry problem on P6.
Z K Y
The post below has been deleted. Click to close.
This post has been deleted. Click here to see post.
Math-wiz
6107 posts
#13 • 1 Y
Y by Adventure10
JND wrote:
Last year, there was also a Geometry problem on P6 of the day 2. Similarly, This year, there is also another geometry problem on P6.

There were 2 options for P6- Algebra or Geometry. I really wished for an Algebra, but P6 is rarely Algebra. So, it was pretty obvious that it would be a Geo
Z K Y
The post below has been deleted. Click to close.
This post has been deleted. Click here to see post.
armpist
527 posts
#14 • 2 Y
Y by Adventure10, Mango247
Dear MLs,

For a variable P on incircle the
locus of Q is Mension circle (BCI),
if it is of any help.

Friendly,
M.T.
Z K Y
The post below has been deleted. Click to close.
This post has been deleted. Click here to see post.
The_Maitreyo1
1039 posts
#15 • 13 Y
Y by Wizard_32, SHREYAS333, Hexagrammum16, RudraRockstar, Purple_Planet, Aryan-23, Hamroldt, TheMathCruncher_007, megarnie, EpicBird08, Adventure10, Mango247, PATRICKLIU
Oh my god feeling soooo good to see an Indian proposing a problem, more specifically a CMI undergrad :gleam:
This post has been edited 1 time. Last edited by The_Maitreyo1, Jul 17, 2019, 1:59 PM
Z K Y
The post below has been deleted. Click to close.
This post has been deleted. Click here to see post.
Dr.Sop
206 posts
#16 • 3 Y
Y by Adventure10, Mango247, ehuseyinyigit
Let $DI$ meet line through $A$ and perpendicular to $AI$ at at $L$. From angle chase $ALDP$ is cyclic. It allows us to make next

Generalization.

$ABC$ given $E, F$ are any points on sides $AC, AB$. $D$ any point. $I$ is the second intersection point of the circumcircles of $DFB$, $DCE$. $L$ is the intersection point of $DI$ and a line through $A$ and parallel to $EF$. $P$ is the second intersection point of the circumcircles of $ALD$, $DEF$. $Q$ is second intersection point of the circumcircles of $PBF, PCE$.

Prove that $P$, $Q$, $L$ are collinear.
Z K Y
The post below has been deleted. Click to close.
This post has been deleted. Click here to see post.
Dr.Sop
206 posts
#18 • 2 Y
Y by buratinogigle, Adventure10
Comment. It is nice problem, and the more general theory of quadric surfaces ( https://en.wikipedia.org/wiki/Hyperboloid ) gives a nice proof of it and better understanding. Actually this theory is a way how I found my generalization at previous post.
This post has been edited 1 time. Last edited by Dr.Sop, Jul 17, 2019, 2:21 PM
Z K Y
The post below has been deleted. Click to close.
This post has been deleted. Click here to see post.
PlsHelp
1 post
#19 • 1 Y
Y by Adventure10
Any solutions using mixtilinear incircles?
Z K Y
The post below has been deleted. Click to close.
This post has been deleted. Click here to see post.
MarkBcc168
1595 posts
#20 • 24 Y
Y by RudraRockstar, Polynom_Efendi, Pi-is-3, AmirKhusrau, Williamgolly, Purple_Planet, Pluto1708, Modesti, Illuzion, Aryan27, math31415926535, thrasher64, alinazarboland, Geometry285, hakN, jeteagle, sabkx, Jalil_Huseynov, Lamboreghini, Adventure10, Mango247, GeoKing, ehuseyinyigit, Kenken49
@above Here is my solution found during the contest. It uses Mixtilinear Incircle extensively.
Lemma 1: $AP$ pass through mixtilinear touchpoint $T$ of $\triangle ABC$.

Proof: Let $R'$ be the reflection of $R$ across $AI$. Clearly $R'$ is the antipode of $D$ w.r.t. $\omega$ so $AR'$ is $A$-Nagel cevian of $\triangle ABC$. Thus $AR$ pass through isogonal conjugate of Nagel point so it passes through $T$.
Lemma 2: $B,I,C,Q$ are concyclic.

Proof: Angle chasing
\begin{align*}
\measuredangle BQC &= \measuredangle BQP + \measuredangle PQC \\
&= \measuredangle BFP + \measuredangle CEP \\
&= \measuredangle FDP + \measuredangle PDE \\
&= \measuredangle FDE \\
&= \measuredangle BIC 
\end{align*}
Extend $PQ$ to meet $\odot(BIC)$ again at $K$. Then note that
$$\angle KBC = \angle KQC = \angle PQC = \angle PEC = \angle PFE$$so $\triangle KBC\sim\triangle PFE$. Further angle chasing gives $\angle RFE = 0.5\angle B$ so $\triangle BKC\cup I\sim\triangle PFE\cup R$. The latter quadrilateral is harmonic so the former is harmonic too. Thus if $N$ be the midpoint of arc $BAC$, then $K\in IN$.

Let $M$ be the other midpoint of arc $BC$ and $T$ be the mixtilinear touch-point. Thus $\angle MTN = 90^{\circ}$ which means $T$ is the midpoint of $IK$. Now, let $S$ be the center of $A$-mixtilinear incircle and $X=DI\cap AN$. So by Lemma 1 and homothety, $IP\parallel ST$. By Menelaus theorem on $\triangle ATN$ , it suffices to show that
$$\frac{AP}{PT}\cdot\frac{TK}{KN}\cdot\frac{NX}{XA}=1$$We compute the first term first, note that
$$\frac{AP}{PT} = \frac{AI}{IS} = \cot^2\frac{\angle A}{2}$$For the second term, we use Power of Point twice.
$$\frac{TK}{KN} = \frac{IT}{KN} = \frac{IT\cdot IN}{NK\cdot NI} = \frac{AI\cdot IM}{NB^2}$$And the last term note that $MN\parallel DI$ so,
$$\frac{NX}{XA} = \frac{MI}{IA}$$Thus multiplying all together, the resulting terms
$$\cot^2\frac{\angle A}{2}\cdot \frac{MI^2}{NB^2} = \cot^2\frac{\angle A}{2}\cdot \frac{MB^2}{NB^2}$$as $\tfrac{MB}{NB} = \tan\tfrac{\angle A}{2}$, we are done.
Attachments:
This post has been edited 2 times. Last edited by MarkBcc168, Jul 23, 2019, 10:49 AM
Z K Y
The post below has been deleted. Click to close.
This post has been deleted. Click here to see post.
duanby
76 posts
#21 • 4 Y
Y by NumberX, supercarry, Adventure10, IMUKAT
Assume J the intersection of DI and ex-anglebisector of A , then it's sufficient to prove the power of J to (BPF),(CPE) are the same, using Casey Theorem, sufficient to prove O_1L= O_2K, where O1, O2 the center of (BPF),(CPE), K,L the midpoint of BI,CI.
The rest is easy, consider triangle IO_1L and IO_2K, with sine law, sin(IO1L)/sin(IO2L) = PF/PE=RF/RE = BI/CI
done
Attachments:
Z K Y
G
H
=
a